For each of the hypothetical data sets, determine whether a bar graph or a pie chart would be an appropriate way to display the
data. In some cases, both types of graphs may be appropriate.
pie chart
the percent of individuals with each of the following natural hair colors:
black, brown, blond, red, and other
bar graph
the land area of each of the approximately 50 countries in Europe
bar graph
the number of college students who play basketball, baseball, or soccer
pie chart
the number of K-12 schools that are public compared to the number of private and
charter schools

Answers

Answer 1

Answer:

See below

Step-by-step explanation:

Pie charts are best to display parts of a whole.Bar graphs are best to compare data.

A. The percent of individuals with each of the following natural hair colors:  black, brown, blond, red, and other

Pie chart

B. The land area of each of the approximately 50 countries in Europe

Pie chart or Bar graph

C. The number of college students who play basketball, baseball, or soccer

Bar graph

D. The number of K-12 schools that are public compared to the number of private and  charter schools

Bar graph

Related Questions

A student has some​ $1 and​ $5 bills in his wallet. He has a total of 14 bills that are worth ​50$. How many of each type of bill does he​ have?

Answers

Answer:

3.5$

Step-by-step explanation:

Nine 5's and five 1's. Nine five-dollar bills hit $45 dollars, and the remaining $5 can only be paid in 1's to reach 14 bills.

what is the slope of the line ?

Answers

The slope of the line is - 3/4

I got my ordered pairs by finding two points on the lines, an noting down their X and Y values.
Then I used the equation Y2 - Y1 (all over) X2 - X1 by plugging in the ordered pairs.
Finally, I solved and got -3/4!
See the attached picture below



Additionally, you can use rise over run, and count the rise and the run. In the image, the rise is three, and the run is four. The slope is negative, which leaves me to know that the answer is -3/4.





Hope this helps :)

A road map is drawn to the scale of 1 inch equals 20 miles. How far apart in miles are two towns which are 2 1/2 inches apart on the map?

Answers

50

Step-by-step explanation:

You set this up with a ratio,  1:20 or 1 equals 20 miles.   So i'm multiplying by 20 regardless.   Now I have 2 and a half inches or you can say 2.5.  Multiply 2.5 (on paper..) and 20 and there you go.    20 times 2 equals 40 and 20 times a half is 10, 40 + 10= 50.  

Find the indicated side of the
right triangle.

X=

Answers

Answer:

tan 30 = p/b

1/√3= 3/x

x= 3√3

please give brainliest as  i am 3 short to expert

Step-by-step explanation:

(8x/2)^3 whats the answer??????

Answers

Answer:

(8x)^(2/3

Step-by-step explanation:

Solve
-6y + 4 = |4y + 12|

Answers

Answer:

[tex]y = - \frac{4}{5} [/tex]

It cost Mila $5.00 to send 125 text messages. What is the cost of sending text messages, in dollars per text?

Answers

Answer:

0.04 dollars per text

Step-by-step explanation:

5 dollars divided by 125 texts is equal to 0.04 dollars per text. SO just do 5/125 to get your answer

answer is 0.04 in text

how do you convert a fraction to decimal

Answers

to convert a fraction to a decimal, divide the numerator by the denominator (divide the top by the bottom)
divide the numerator by the denominator. ex) 1/2 divide 1 by 2 to get an answer of .5

10. The weight of an object on a particular scale is 125.2 lbs. The measured weight

may vary from the actual weight by at most 0.4 lbs. What is the range of actual

weights of the object?

A. 124 8 2 x 2 125.6

B. 124.8 2 x 125.6

C. 124 8 5 x 125.6

D. XS 124.8 or x 21256

Answers

Given :

The weight of an object on a particular scale is 125.2 lbs.

The actual weight by at most 0.4 lbs.

To Find :

The range of actual  weights of the object.

Solution :

It is given that uncertainty in weight is 0.4 lbs.

So, minimum weight can be( 125.2 -0.4 ) = 124.8 lbs.

Maximum weight is given by  ( 125.2 + 0.4 ) = 125.6 lbs.

Therefore, the range of actual  weights of the object is between :

124.8  x 125.6 .

Hence, this is the required solution.

help me please. 15 POINTS BRAINLEST ANSWER EVER (y+4)–(y–1)=6y

Answers

Answer:

5/6

Step-by-step explanation:

y+4-y+1=6y

5=6y

y=5/6

Solution for (y+4)-(y-1)=6y equation:

Simplifying

(y + 4) + -1(y + -1) = 6y

Reorder the terms:

(4 + y) + -1(y + -1) = 6y

Remove parenthesis around (4 + y)

4 + y + -1(y + -1) = 6y

Reorder the terms:

4 + y + -1(-1 + y) = 6y

4 + y + (-1 * -1 + y * -1) = 6y

4 + y + (1 + -1y) = 6y

Reorder the terms:

4 + 1 + y + -1y = 6y

Combine like terms: 4 + 1 = 5

5 + y + -1y = 6y

Combine like terms: y + -1y = 0

5 + 0 = 6y

5 = 6y

Solving5 = 6y

Solving for variable 'y'.

Move all terms containing y to the left, all other terms to the right.

Add '-6y' to each side of the equation.

5 + -6y = 6y + -6y

Combine like terms: 6y + -6y = 0

5 + -6y = 0

Add '-5' to each side of the equation.

5 + -5 + -6y = 0 + -5

Combine like terms: 5 + -5 = 0

0 + -6y = 0 + -5

-6y = 0 + -5

Combine like terms: 0 + -5 = -5

-6y = -5

Divide each side by '-6'.

y = 0.8333333333

Simplifying

y = 0.8333333333

Rhonda had $300 to buy supplies for the animal shelter. She bought 8 bags of dog food and 11 leashes. Each bag of food cost $25.19, and each leash cost $4.65. How much money did Rhonda have left after her purchases? $11.65 $14.29 $22.91 $47.33

Answers

Hey there! I'm happy to help!

We have 8 bags of dog food and each costs $25.19

8(25.19)=201.52

And we have 11 leashes each costing $4.65.

11(4.65)=51.15

We add these totals up.

201.52+51.15=252.67

We subtract this from 300 to see how much is left.

300-252.67=47.33

Rhonda had $47.33 left after her purchases.

Have a wonderful day! :D

Answer:

$47.33

Step-by-step explanation:

she bought 8 bags of dog food so: 8 x 25.19 = 201.52

she bought 11 leashes so: 4.65 x 11 = 51.15

201.52 + 51.15 = 252.67

300 - 252.67 = 47.33

Suppose that the overall probability of living to age 70 is 0.62 and that the overall probability of living to age 80 is 0.23. If a person reaches their 70th birthday, what is the probability that they will live to age 80?

Answers

Given :

P( X = 70 ) = 0.62 .

P( X = 80 ) = 0.23 .

To Find :

We need to find If a person reaches their 70th birthday, what is the probability that they will live to age 80 .

Solution :

We need to find :

[tex]P(X=70 | X=80 )[/tex]

We know ,

[tex]P(x,y ) = \dfrac{P(x)}{P(y)}[/tex]

So ,

[tex]P(X=70 | X=80 )=\dfrac{P(X=80)}{P(X=70)}\\\\=\dfrac{0.62}{0.23}=0.371[/tex]

Hence , this is the required solution .

i’m stuck on this question please help me

Answers

Answer:

im pretty sure its the 2nd or 3rd one

dives from a platform that is 5 meters above water. dive takes her 2.5 below the surface of the water. How far does ​'s dive take ​?

Answers

Answer:

7.5 meters

Step-by-step explanation:

Stay safe!!! Stay 6 feet!!

Answer:

its 7.5

i got it right

How many 1/6s are in 2/3?

Answers

Answer:

four

Step-by-step explanation:

there are 4 1/6s in 2/3. 1/6 times four is 4/6. divide numerator and denominator by two to simplify, and you get 2/3. I hope this is correct :)

The number 1/6 s in the fraction 2/3 are 4.

The given fractions are 1/6 and 2/3.

Dividing fractions is nothing but multiplying the fractions by reversing one of the two fraction numbers or by writing the reciprocal of one of the fractions. By reciprocal we mean, that if a fraction is given as a/b, then the reciprocal of it will b/a.

Number of 1/6s are in 2/3

= 2/3 ÷ 1/6

= 2/3 × 6/1

= 4

Verification:

1/6 + 1/6 + 1/6 + 1/6

= 4/6

= 2/3

Therefore, the number 1/6 s in the fraction 2/3 are 4.

Learn more about the division of fractions here:

https://brainly.com/question/17205173.

#SPJ6

537²×15.90²=74²

I know this looks like pythagorean theorem. But how do they teach this to high schooler?​

Answers

Answer:

It looks like this question is not solvable. So, if this is absolutely correct, write "no solution".

Step-by-step explanation:

Below is the first step to attempt solving the equation.

The little twos at the top of each number are called exponents. This means a number times itself.

So,

537*537=288,369

15.9*15.9=252.81

74*74=5476

Now, take the 288,369*252.81 to get a total of 72,902,566.89 .

Since this answer is (obviously) not equal to the 5476 on the other side, the two are not equal.

Here's how you'd work it out on paper to show it isn't solvable.

(537*537)*(15.9*15.9)= (74*74)

(288,369)*(252.81)= 5476

72,902,566.89 ≠5476

So the answer would be:

Not solvable.

what is the value of x in the proportion ​

Answers

Answer:

  5 2/5

Step-by-step explanation:

Multiply by (3 3/5)x/(1 1/2):

  x = (2 1/4)(3 3/5)/(1 1/2) = (9/4)(18/5)(2/3) =27/5

  x = 5 2/5

Height for each toddler at Mrs.orlando daycare

Answers

1=81 centimetres
1=82 centimetres
2=83 centimetres
3=84 centimetres
1=85 centimetres

Answer: 1.5

Step-by-step explanation:

To find the quartiles, let's first find the median by sorting the data points from least to greatest.

81

,

82

,

83

,

83

84

,

84

,

84

,

85

81,82,83,83⋮84,84,84,8581, comma, 82, comma, 83, comma, start color gray, 83, end color gray, \varvdots, rectangle, start color gray, 84, end color gray, comma, 84, comma, 84, comma, 85

The median is the mean of the two middle numbers.

83

+

84

2

=

167

2

=

83.5

2

83+84

=

2

167

=83.5start fraction, start color gray, 83, end color gray, plus, start color gray, 84, end color gray, divided by, 2, end fraction, equals, start fraction, 167, divided by, 2, end fraction, equals, start color #9d38bd, 83, point, 5, end color #9d38bd

The first quartile is the median of the data points to the left of the median.

81

,

82

,

83

,

83

81,82,83,8381, comma, 82, comma, 83, comma, start color gray, 83, end color gray

Q

1

=

82

+

83

2

=

165

2

=

82.5

Q

1

=

2

82+83

=

2

165

=82.5Q, start subscript, 1, end subscript, equals, start fraction, 82, plus, 83, divided by, 2, end fraction, equals, start fraction, 165, divided by, 2, end fraction, equals, start color #ff00af, 82, point, 5, end color #ff00af

The third quartile is the median of the data points to the right of the median.

84

,

84

,

84

,

85

84,84,84,85start color gray, 84, end color gray, comma, 84, comma, 84, comma, 85

Q

3

=

84

+

84

2

=

168

2

=

84

Q

3

=

2

84+84

=

2

168

=84Q, start subscript, 3, end subscript, equals, start fraction, 84, plus, 84, divided by, 2, end fraction, equals, start fraction, 168, divided by, 2, end fraction, equals, start color #6495ed, 84, end color #6495ed

IQR

=

Q

3

Q

1

=

84

82.5

=

1.5

IQR

=Q

3

−Q

1

=84−82.5

=1.5

HELP PRECALC DO NOT UNDERSTAND WILL GIVE BRAINLIEST

Answers

Answer:

  the lower right matrix is the third correct choice

Step-by-step explanation:

Your problem statement shows that you have correctly selected the matrices representing the initial problem setup (middle left) and the problem solution (middle right).

Of the remaining matrices, the upper left is an incorrect setup, and the lower left is an incorrect solution matrix.

__

We notice that in the remaining matrices on the right that the (2,3) term is 0, and the (3,2) and (3,3) terms are both 1.

The easiest way to get a 0 in the 3rd column of row 2 is to add the first row to the second. When you do that, you get ...

  [tex]\left[\begin{array}{ccc|c}1&1&1&29000\\1+2&1-3&1-1&1000(29+1)\\0&0.15&0.15&2100\end{array}\right] =\left[\begin{array}{ccc|c}1&1&1&29000\\3&-2&0&30000\\0&0.15&0.15&2100\end{array}\right][/tex]

Already, we see that the second row matches that in the lower right matrix.

The easiest way to get 1's in the last row is to divide that row by 0.15. When we do that, the (3,4) entry becomes 2100/0.15 = 14000, matching exactly the lower right matrix.

The correct choices here are the two you have selected, and the lower right matrix.

What information can be used to compare linear relationships. Explain why

Answers

Step-by-step explanation:

y=mx+b m being the highest point x being the lowest

Answer:

Linear relationships can be compared using their initial values, or y-intercepts, and their rates of change, or slopes. Initial values can tell you which relationship started with a greater value. Comparing slopes can tell you which relationship is rising or falling faster.

Step-by-step explanation:

This is the sample response.

HELP WILL GIVE BRAINLIEST!!Aiden has $15.00 on his copy card. Each time he uses the card to make a photocopy, $0.06 is deducted from his card. Aiden wants to be sure that there will be at least $5.00 left on his card when he is finished. The inequality below relates x, the number of copies he can make, with his copy card balance.

15 minus 0.06 x greater-than-or-equal-to 5

What is the maximum number of copies Aiden can make?

Answers

Answer:

166

Step-by-step explanation:

$15.00-$5.00= $10.00 (subtract five dollars because aiden wants to have at least five dollars left)

10.00 divided by 0.06=  166

I hope this helps! :)

AN = 16CM; AC = 18CM & CN = (4X -6) CM. Find the value of X

Answers

Answer:

x = 6

Step-by-step explanation:

Since <A is congruent to <N, they are both an isosceles base angles. ∆CAN is an isosceles ∆.

Therefore, one of the properties of an isosceles ∆ is that two sides are equal, aside the base of the isosceles. This implies that: CN = AC

AC = 18cm

CN = (4x - 6)cm, therefore, we can generate an equation for solve for x as follows,

4x - 6 = 18 (two sides of an isosceles ∆ are congruent)

Add 6 to each side of the equation.

4x - 6 + 6 = 18 + 6

4x = 24

Divide both sides by 4

4x = 24/4

x = 6

find 6/7 divided 2/9. and put your answer as a reduced fraction.​

Answers

Answer:

3 6/7

Step-by-step explanation:

divide 6/7 by 2/9 and that equals 3 6/7 and since you cant simplify that the answer stays the same which the answer is 3 6/7

PLEASE HELP FAST...The dot plots show the numbers of children’s books purchased by customers at two different bookstores on one day which statement is supported by the information in the dot plots

Answers

Answer:

The mode of the data for Store 2 is greater than the mode of the data for Store 1.

Step-by-step explanation:

The mode of store 2 greater than mode of store 1.

What is box- plot?

A box plot, commonly referred to as a box and whisker plot, is a form of chart that is frequently used in explanatory data analysis in descriptive statistics. Box plots use the data's quartiles (or percentiles) and averages to visually depict the distribution of numerical data and skewness.

Given:

Dot plots show the numbers of children’s books purchased by customers at two different bookstores.

Now, from the given figure

The mode of store 1 is 3

and, mode of store 2 is 4.

Hence, the mode of store 2 greater than mode of store 1.

Learn more about box- plot here:

https://brainly.com/question/1523909

#SPJ6

In a sample fruit basket, there are 2 apples for every 3 oranges.
To make a large fruit basket with the same ratio using 20 apples, how many oranges are needed?


Help me please?!

Answers

Answer:

B 32

Step-by-step explanation:

30 oranges would be required to maintain the same ratio with 20 apples.

Solving linear equation.

In the example fruit basket, there are 2 apples for every 3 oranges, based on the provided ratio.

We must establish a proportion in order to determine how many oranges are required to keep the same ratio with 20 apples.

Let "x" stand for the required quantity of oranges.

The ratio can be configured as follows:

20 apples/x oranges = 2 apples/3 oranges.

Cross-multiplication allows us to find the value of "x":

3(20) = 2x

2x = 60

x = 30

As a result, 30 oranges would be required to maintain the same ratio with 20 apples.

Learn more on linear expression here:  https://brainly.com/question/2030026

#SPJ2

I need help on this question 109^41/109^94

Answers

Answer:

[tex]\frac{1}{109^{53}}[/tex]

Step-by-step explanation:

[tex]\frac{109^{41}}{109^{94}}=109^{41-94}=109^{-53}=\frac{1}{109^{53}}[/tex]

100 points HELP / what is the slope of the line in the graph

A Negative four-thirds
B Negative three-fourths
C Three-fourths
D Four-thirds

Answers

B) Negative 3/4

The slope of a line is defined as:

Answer:

It is B

Step-by-step explanation:

Solve the following proportion for u.
u/4 = 3/17
Round your answer to the nearest tenth

Answers

Answer:

345897

Step-by-step explanation:

423786945

I really need helpp ​

Answers

Answer:

C

Step-by-step explanation:

Answer: B. Sandwich Town's Dipping Sauce

Step-by-step explanation: Sandwich town has 1/2 more spoonful than Burger Barn so Sandwich Town has a stronger mustard flavor.

Hope this helps! : )

The telephone company offers two billing plans for local calls. Plan 1 charges ​$32 per month for unlimited calls and Plan 2 charges ​$17 per month plus ​$0.04 per call.
a. Use an inequality to find the number of monthly calls for which Plan 1 is more economical than Plan 2.
b. Explain the meaning of the answer to part a.

Answers

Step-by-step explanation:

Answer in image below press on it

Other Questions
Use the quadratic formula, x=bb24ac2a, to solve the equation 2x26x+3=0. Match each step to the correct method for solving the equation. The graph shows the kinetic energy of both a school bus and a pickup truck as they move at various speeds. Under which of the following conditions can the pickup truck have more kinetic energy than the school bus? The pickup truck is moving at 1 m/s and the school bus is moving at 2 m/s. The pickup truck is moving at 4 m/s and the school bus is moving at 2 m/s. The pickup truck is moving at 2 m/s and the school bus is moving at 3 m/s. The pickup truck is moving at 4 m/s and the school bus is moving at 3 m/s. 3Which expression is closest in value to 5.66? A chef turned on a faucet and filled a 2-gallon pot in 5 6 of a minute. What is the rate of water flow? Simplify your answer and write it as a proper fraction, mixed number, or whole number. The formula d = rt gives the distance, d, traveled in time, t, at rate, r. A motorcycle traveled 67.2 mi in 1.2 h. What was its average speed (rate)? 30 mph 56 mph 66 mph 80 mph Please help. The tiger lost 9 games won 7 games and tied 2 games what percent of the games did they win. What is 78,652 divided by 8 long division What is the name of the system that standardizes measurements in science The hand is located at what end of the forearm Interior Distal Proximal Medial If you flip a coin ten times and get tails three times; what is the experimental probability of getting heads? Sara received 73 votes, Ben received 23 fewer than Sara. How many students voted? How many votes did Ben receive? If 2x=4(52x), then x = when used to treat eating disorders, _____ helps patients develop more realistic body ideals. Lana uses 0.2 pounds of peaches in each mini pie she makes. How many mini pies can she make with 2.50 pound of peaches? Can someone please help out with this? I have 0 clue what Im doing and it doesnt seem too difficult, thanks! Solve for x in simplest form. 7=8/5 (3x+5) Which is the best example of outsourcing? A US shoe company opens a factory in China and hires Chinese workers to make shoes. A US shoe company opens a factory in the US and hires US workers to make shoes. A US shoe company opens a factory in the US and hires Chinese workers to make shoes. A Chinese shoe company opens a factory in China and hires Chinese workers to make shoes. Why would a nation want to creat a colony What is the average speed of a motorcycle that goes 90 meters in 5 seconds? 4. A mistake was made when solving the problem below.Line 1: 2022+6Line 2: 204 + 6Line 3: 80 + 6Line 4: 86The mistake was made in Line